Normalización de la integral de trayectoria

Cuando uno define el propagador integral de trayectoria, existe la necesidad de normalizar el propagador (ya que le daría una densidad de probabilidad). Hay dos fórmulas que se utilizan.

1) Original (v1+v2) : La primera fórmula (con la que puedo estar de acuerdo intuitivamente) dice que:

(1) D X b d X b | k ( X b t b | X a t a ) | 2 = 1

para todos los valores de X a en valores fijos de t a , t b y donde D X b significa el dominio de X b .

1') Actualización (v3+v4) : cambié de opinión (para estar más de acuerdo con las reglas de Born). La primera fórmula (con la que puedo estar de acuerdo intuitivamente) dice que:

(1') | D X b d X b k ( X b t b | X a t a ) | 2 = 1

para todos los valores de X a en valores fijos de t a , t b y donde D X b significa el dominio de X b .

2) La segunda fórmula (que en realidad también es muy intuitiva) dice que:

(2) límite t b t a k ( X b t b | X a , t a ) = d ( X b X a ) .

Ahora, estos generalmente se tratan como equivalentes, pero no puedo ver directamente cómo puede ser este el caso. ¿No es la segunda fórmula menos restrictiva?

Respuestas (2)

I) Conceptualmente, la ecuación original de OP. (1)

(1) R d X F   | k ( X F , t F ; X i , t i ) | 2   = ?   1 ( ¡Equivocado! )

choca (como OP se dio cuenta de forma independiente) con el principio fundamental de la integral de trayectoria de Feynman de que la amplitud

k ( X F , t F ; X i , t i )   =   h i s t .

es una suma de historias, mientras que la probabilidad

PAG ( X F , t F ; X i , t i )   =   | k ( X F , t F ; X i , t i ) | 2     h i s t .

no es una suma de historias.

Concretamente, el fallo de la ec. (1) también puede verse como sigue. Si asumimos que 1

(A) k ( X i , t i ; X F , t F )   =   k ( X F , t F ; X i , t i ) ¯ ,

y la propiedad de (semi)grupo de los propagadores/núcleos de Feynman

(B) k ( X F , t F ; X i , t i )   =   R d X metro   k ( X F , t F ; X metro , t metro ) k ( X metro , t metro ; X i , t i ) ,

entonces el lhs. de la primera ecuación original de OP. (1) con ( X i , t i ) = ( X F , t F ) no es igual a 1 , pero en cambio se vuelve infinito

(C) k ( X F , t F ; X i , t i )   =   d ( X F X i )   =   d ( 0 )   =   , X i   =   X F , t i   =   t F ,

debido a la segunda fórmula de OP (2).

II) El resultado de la normalización infinita (C) puede entenderse intuitivamente de la siguiente manera. Recuerde que las trayectorias en la integral de trayectoria satisfacen la condición de contorno de Dirichlet X ( t i ) = X i y X ( t F ) = X F . En otras palabras, la partícula está localizada en X -colocar el espacio en tiempos inicial y final. Por otro lado, una partícula localizada en X -el espacio de posición corresponde a una función de onda función delta Ψ ( X ) = d ( X X 0 ) , que no es normalizable, cf. por ejemplo , este y este Phys.SE publicaciones.

III) Conceptualmente, la primera eq. de OP. (1')

(1') | R d X F   k ( X F , t F ; X i , t i ) |   = ?   1 ( ¡Resulta estar en última instancia equivocado! )

es la afirmación de que una partícula que se localiza inicialmente en un evento de espacio-tiempo ( X i , t i ) debe con probabilidad 100% estar dentro X -espacio R en un momento final t F , ya que nuestro modelo QM no permite la creación o aniquilación de partículas. Sin embargo, tal noción de probabilidades absolutas del kernel de Feynman k ( X F , t F ; X i , t i ) no se puede mantener cuando un concepto tiene que convertirse en fórmulas matemáticas, como se analiza en detalle en esta publicación de Phys.SE. En general, la primera ecuación de OP. (1') solo se mantiene por tiempos cortos Δ t τ , dónde τ es una escala de tiempo característica del sistema.

IV) Ejemplo. Finalmente, consideremos el ejemplo de una partícula libre no relativista en 1D. El propagador de Feynman luego lee

(D) k ( X F , t F ; X i , t i )   =   A π mi A ( Δ X ) 2   =   metro 2 π i 1 Δ t Exp [ i metro 2 ( Δ X ) 2 Δ t ] , A   :=   metro 2 i 1 Δ t , Δ X   :=   X F X i , Δ t   :=   t F t i     0.

[Es un ejercicio instructivo mostrar que la fórmula (D) satisface las ecs. (AC) y la segunda fórmula de OP (2).] La integral de Gauss sobre X F es uno

(MI) R d X F   k ( X F , t F ; X i , t i )   =   1 ,

lo que muestra que la primera ecuación de OP. (1') en realidad se cumple para una partícula libre. el integrando

(F) | k ( X F , t F ; X i , t i ) | 2   =   | A | π   =   metro 2 π 1 | Δ t | , Δ t     0 ,

en la izquierda. de la primera ecuación original de OP. (1) es independiente del punto medio X metro . Por lo tanto, la integral sobre X metro (es decir, a la izquierda de la primera ecuación de OP (1)) se vuelve infinito

(GRAMO) R d X F   | k ( X F , t F ; X i , t i ) | 2   =   metro 2 π 1 | Δ t | R d X F   =   , Δ t     0 ,

de acuerdo con lo encontrado en la ec. (C) en la sección I.

Referencias:

  1. RP Feynman y AR Hibbs, Quantum Mechanics and Path Integrals, 1965.

--

1 Tenga en cuenta que la ref. 1 define k ( X F , t F ; X i , t i ) = 0 si t i > t F , ver ref. 1 entre ec. (4-27) y ec. (4-28). Aquí asumimos la propiedad (A) en su lugar.

Sí, claro ! Disculpe, me atrevo a generalizar demasiado rápido, de hecho, de acuerdo con la segunda ecuación, ¡esto debería cumplirse! Así que también debería exigir eso como t a t b eso X a X b ¡porque mi función delta de hecho explotaría! ¿O este razonamiento no es correcto?
Ajusté mis ecuaciones, no sé si parecen correctas ahora. Pero creo que la primera ecuación exige que el propagador se normalice para valores fijos de tiempo que no son iguales y que la segunda ecuación exige que para valores fijos de espacio se mantenga el límite.
@Qmechanic, nuevamente gracias por la gran actualización. Por supuesto, las fórmulas exactas también pueden derivarse de los libros de Kleinert y Schulman. Su derivación era menos de mi preocupación en realidad (aunque muy educativo, por supuesto). Para mí, se trataba más de la cuestión de si se necesitan ambos. ¡Por supuesto, la analogía con la mecánica cuántica muestra esto de una manera hermosa! Tal vez hubiera esperado argumentos que no usaran nada de la mecánica cuántica, ¡pero esto servirá! :)
Actualicé la respuesta.
@Cosmas Zachos: ¡Gracias!

Su primera fórmula es incorrecta. Esta distribución no puede normalizarse. Solo podemos obtener distribuciones de probabilidad relativa del cuadrado absoluto del kernel. Tiene un factor de normalización, pero este es un factor diferente, este factor se relaciona con la definición de la integral de trayectoria. Consulte la sección 4.1 en 'Integrales de trayectoria en mecánica cuántica' de Feynman para comprender cómo se obtiene este factor. Sabemos

D X b k ( X C t C | X b t b ) d X b k ( X b t b | X a t a ) = k ( X C t C | X a t a )
dónde t C > t b > t a

En tu segunda fórmula t b > t a , por lo que el límite es un límite por la izquierda.

Aplicando el límite de t C t a a la segunda integral que obtenemos (que debería haber sido tu primera fórmula)

D X b k ( X C t C | X b t b ) d X b k ( X b t b | X a t a ) = d ( X C X a )

Así podemos mostrar, en el límite t C t a

k ( X C t C | X a t a ) = d ( X C X a )

El valor absoluto de los propagadores de Feynman multiplicado por d X C le dará una probabilidad relativa y no una probabilidad exacta. Esta es la razón por la cual la integral en tu ecuación debe divergir. Si lo observable X tomó un conjunto de valores finitos X 1 , . . . . , X norte , entonces reemplazaríamos la integral con una simple suma y obtendríamos el mismo límite:

Σ X i k ( X metro t C | X i t b ) k ( X i t b | X norte t a ) = d metro norte

Entonces, ¿realmente se necesitan ambas demandas? El primero nos dice algo sobre la normalización global , el segundo nos dice algo sobre *la fase del propagador y cómo debería comportarse cuando t b t a , y por supuesto el centrado ya que está describiendo partículas y queremos que evolucionen de forma continua. ¿Es esto más correcto entonces?
Sí, pero actualmente estoy tratando de seguir un camino en el que definimos el camino integral y la mecánica cuántica sin usar Schrödinger de ningún conocimiento cuántico.
De hecho, su línea de razonamiento tiene mucho sentido para mí si tuviera que pasar de los resultados conocidos en la mecánica cuántica a una formulación de integral de trayectoria. Ahora ajusté mi pregunta ayer para hacer que la primera fórmula sea más correcta (había publicado mal). Lo que intenta el texto que estoy leyendo es formular la mecánica cuántica de tal manera que esté en la misma línea que la mecánica clásica (principio de "mínima" acción) SIN conocimientos previos de mecánica cuántica. Empezamos con el experimento de las 2 rendijas e imponemos la regla de Born. Esa regla establece que debemos sumar todos los caminos (fin de la parte 1)
(parte 2) esto nos da la idea de la integral de camino (de hecho, esto es lo que dice Feynman al sumar todos los caminos). Ahora puedo escuchar tu siguiente pregunta: "Está bien, entonces, ¿cómo determinas el 'peso' de un camino?". Bueno, junto con las reglas de nacimiento, simplemente establecemos que el peso (o la fase de lo que sea) debe tener la forma mi X pag ( i S ) , donde i es para convertirlo en una fase (para que podamos obtener los efectos de interferencia que están presentes en el experimento de doble rendija), es un número muy pequeño que exige que (según el principio de acción mínima) (fin de la parte 2)
(parte 3), deberíamos tener d S = 0 , ahora desde 0 , la demanda con esta fase se convierte en d s 0 . Y en el límite clásico, donde S >> esto realmente da d S = 0 . Así que hemos definido una fase que hace una transición perfecta. Sé que puedes derivar esto de la mecánica cuántica con el operador de evolución temporal, pero esa es una de las cosas que NO se usan. Pero para terminar, me gusta mucho tu comentario ya que me ha dado una idea del vínculo entre la ortogonalidad de la función de onda y mi segunda fórmula. Revisaré mi Feynman también. ¡gracias!